This is a strengthening question, as we are asked which answer choice: if true, provides the most support for the argument?
The stimulus begins by informing us that there are two subsets of a particular species of bird; crested and non-crested. The birds usually live in flocks composed of only one of these subsets, and have a preference for birds of the same variety as themselves. So in a crested flock you are likely to find only crested birds, and these birds will likely select crested mates. Interesting! We are further told that even if you move a bird from a flock where all the other birds are crested into a mixed flock, it will select a crested mate regardless of whether it itself is crested or non-crested. It’s important in this sentence that we realize that a flock where all other birds are crested does not mean the bird in question is itself crested. So it seems like non-crested birds from crested flocks will go against the general trend of the birds selecting mates of the same type. The author concludes from this that the preference for mates is learned rather than genetically determined.
If we think about the author’s reasoning, it does make some sense. We would expect birds to mostly select the same kind of mate if preference was learned since flocks are usually composed of one type, and this would also explain why there are exceptions when a bird is raised in a flock mostly of the other type. Since we are choosing one hypothesis (learned) over another (genetic) we should look for answers that support the learned hypothesis over the genetic one. Let’s look at our options:
Answer Choice (A) OK? First of all we are not interested in other bird species, we are interested in why this particular species has a pattern in its preferences for specifically whether a mate is crested or non-crested.
Answer Choice (B) Interesting! But we want support for why a particular behavior (mate selection) is caused by nurture rather than nature.
Answer Choice (C) We want to know why they have a particular preference, not whether they have other preferences.
Answer Choice (D) This actually weakens our learned trait hypothesis, because it shows that a bird can have a mating preference if there was no opportunity to have learned it since the bird was raised in captivity.
Correct Answer Choice (E) This strengthens our learned hypothesis by showing that in the absence of a crested or non-crested homogenous flock, birds lack the mating preference. This is what we would expect if the preference was learned, and not expect if it was genetic.
This is a very standard NA question stem. If the argument depends on an assumption, the assumption is necessary.
So the plastic rings on a six-pack ensnare and suffocate animals. Yeah, I’ve definitely heard this and always try to cut these up before discarding. I don’t know if it actually helps, but can’t hurt. Anyway, what about it? Oh okay, new rings that will disintegrate after three days of sunlight. A few things about this sentence. First, is this going to work? I mean, it seems like these might get this much light just in shipping and handling before their job is done. This does not matter though. Despite any potential downside, this statement tells us that ALL beverage companies will soon be using these rings. That seems like a pretty big shift. Good. And once we complete the switch to the new rings, the threat plastic rings pose to wildlife will be eliminated. So this is the conclusion.
I can see a few possible problems here, but we don’t want to go hunting for anything specific. For NA questions, it is best to go into the answer choices with an open mind. Let them speak to you and consider the issues they suggest.
Answer Choice (A) Does this have to be true for the argument to work? No. What if some of them will disintegrate in only two days? Well that might be a problem as far as these rings efficacy as packaging, but our conclusion is only about eliminating threat to wildlife. If three days is good, two seems like it would be even better. So when we negate this, it seems to only make things better for our wildlife friends.
Answer Choice (B) We don’t care about this at all. What matters is that these companies are making the switch. End of story. This can bankrupt them for all we care.
Correct Answer Choice (C) So if this isn’t true, has the threat been eliminated? I would have to say it has not. Three days seems like fast disintegration time, but now that this answer directs my attention to it, the conclusion is quite strong. Our argument is claiming to eliminate the threat. Elimination is as absolute as it gets. But these things have up to three days to be out there before they fall apart. And that’s enough time to harm some wildlife. So now it seems the threat is not eliminated without this answer choice. The argument does, indeed, require this to be true. So this is our answer.
Answer Choice (D) Interesting suggestion, but this is not necessary. If we negate this, we’re only expanding the harms being caused by the old rings. But the conclusion is limited to eliminating the threat of suffocation—not all possible threats—so even if there are other threats, they are not relevant to our argument.
Answer Choice (E) Well I certainly hope not. Regardless, does this have to be true? No. We care about the threat of suffocation which remains a threat even if some animals are able to escape the rings. A threat need not be universally fatal to qualify as a threat. A 99% mortality rate would still be quite threatening.
Answers D and E may be tempting because they address the old rings. There is no reason to believe that the old rings will immediately disappear from the environment. They may continue posing a threat even after their use is discontinued in new packaging. So there is an additional necessary condition for this argument which concerns eliminating the risk posed by the old rings, but this answer does not express that assumption correctly. This alternative necessary assumption is a likely pre-phrase, and the possible existence of multiple necessary assumptions is why this strategy is not recommended for the question type. If you D or E based on a pre-phrase, ask yourself: Did you really scrutinize answer choice C to consider what it was suggesting? Or did you dismiss it because it didn’t match your expectation?
Here we have a strengthening questions, since the question stem demands: Which one of the following, if true, most strongly supports the conclusion drawn from the survey results?
The stimulus starts with the claim that financial success does not guarantee happiness. We should pause here and think about what a guarantee means. If something guarantees something else, then it is sufficient for something else. So what this first sentence really means is that it is not true that if you are financially successful then you are happy; financial success is not sufficient to produce happiness. The next sentence reveals the author’s position, as they assert that this claim has been verified by statistics. Interesting! Since this is a strengthening question, we want to anchor ourselves in this conclusion and get a good sense of what is supporting it. The final sentence introduces the support and tells us about a recent survey, where only one-third of respondents who claimed financial success also reported that they were happy.
If financial success did guarantee happiness, we would expect 100% of people with financial success to report that they are happy. The fact that only 33% did so is good evidence for our author’s conclusion. Unfortunately, there is just one problem. If we read the last sentence carefully, our sample for this statistic is composed of people who claimed to have financial success. Self-reporting should never inspire confidence. While there are many ways to strengthen this argument, we should be on the lookout for an answer that strengthens the connect between the respondents and being truly financially successful. Let’s see what the answers have in store for us:
Correct Answer Choice (A) This does exactly what we noticed the support needed. It strengthens the connection between our sample and actual financial success. If this is true, it guarantees there are at least some people who are financially successful and not happy, and therefore it is impossible that financial success is sufficient for happiness.
Answer Choice (B) We don’t care what people used to think or do think, nor whether financial success is necessary to happiness. Our conclusion is that, regardless of opinion, financial success is not sufficient for happiness.
Answer Choice (C) Financial security being sufficient for happiness wouldn’t mean that it is necessary to have financial security to be happy, so the existence of happy people without financial security isn’t strong support for our position. Even worse, this study still has the problem of self reporting.
Answer Choice (D) We are only told about what proportion of respondents who did report financial success were happy, we don’t know anything about the respondents who didn’t. Even if they were financially successful, maybe the all reported they were happy, in which case our argument would actually be weaker. Not enough information here.
Answer Choice (E) This would weaken our support by making a larger portion of those who reported financial success actually be happy.
This is a pseudo-sufficient assumption question because of the stem: Which one of the following principles... supports the view that a physician’s responding to the request would violate medical ethics? This is at once a PSAr question and a
The stimulus tells us that the police have a wanted poster in a medical journal for a specific fugitive with a non-infectious skin condition that would eventually require him to visit a doctor, asking for information on where the fugitive is. Because doctors are required to report gunshot wounds to police and certain infectious disease to the health authorities, reporting information about this fugitive, according to the argument, would not violate medical ethics. Our author concludes that these exceptions to confidentiality, meaning sharing information on our fugitive, are clearly ethical.
if we read those requirements again very closely, we’ll see that the conclusion does not follow logically from this. Based on the requirement (report to police if it’s a gunshot wound OR health authorities if it’s certain types of infectious diseases), the physician in our stimulus shouldn’t report our fugitive since he has neither a gunshot wound nor an infectious disease.
Figuring out what the question stem is asking us to do is extremely important here. Since our question stem is essentially asking us to support a view that disagrees with the stimulus, our correct answer choice will weaken the argument that the doctor is not violating medical ethics. In other words, we’re trying to weaken the idea that a doctor reporting to the authorities with information on the fugitive is an ethical exception to confidentiality.
Answer Choice (A) This rule brings forward requirements as citizens to report certain issues to the authorities, and nothing about the doctor violating medical ethics. This doesn’t weaken our conclusion by way of a rule/assumption.
Answer Choice (B) We still have to observe the requirements put forward in the stimulus; this answer choice would go against our requirements.
Correct Answer Choice (C) This is correct because it’s pointing out that unless the fugitive had a gunshot wound (thereby requiring the doctor to report the fugitive to the police), the doctor should not be sharing information on the fugitive with anyone (which would then reduce their willingness to come in to be treated for fear of being detained).
Answer Choice (D) Similar to answer choice (B), this would go against the requirement laid out in the stimulus.
Answer Choice (E) Again, we are given exceptions to confidentiality (gunshot wounds and infectious diseases), so this answer choice goes against this requirement, much like answer choices B and D do.
This is a sufficient assumption question, as indicated by the question stem: The conclusion above is properly drawn if which one of the following is assumed?
This is a very conditional-heavy stimulus.
Premise 1: /strike → increase wages
Premise 2: increase wages → sell sub
Conclusion: sell sub
Remember, conditionals have to be triggered in order for us to conclude anything from them. Since we’re trying to prove that Bell’s subsidiaries will be sold, we need something to trigger the first or second premise: either that there will not be a strike or that the wages will be increased.
Answer Choice (A) This does have anything to do with our premise or conclusion and it does help validate our conclusion. It doesn’t trigger our conditionals either.
Answer Choice (B) This is negating the necessary condition in the first premise, which would trigger backward, allowing us to conclude that the workers will do on strike. This answer choice also negates the sufficient condition in premise 2, which renders the rule useless. This is incorrect.
Correct Answer Choice (C) This triggers the first premise, which also triggers our second premise, leading to the conclusion that Bell’s subsidiaries will be sold.
Answer Choice (D) This AC is saying the president can increase wages; this does not mean that he will.
Answer Choice (E) This is additional information that does not clearly trigger our conditionals. The workers only wanting a wage increase does not help us validate the conclusion.
This is a Method of Reasoning question, and we know this because of the question stem: “The argument proceeds by..”
The argument starts out by saying that solutions to reduce healthcare costs can’t be found with our current system. Why? Well, because each party is trying to shift the costs onto the next party, including the patients. The author also says this is exactly what happened next in the 1980s reforms, and provides a visual explanation of what happened when we try to cut costs somewhere; it pops up in the same amount elsewhere. Then, he cites an example of this: the government tried to prevent patients from getting certain costs covered and then they had worse conditions afterward.
Lots happening here! The conclusion is the first sentence; everything laid out after lends support to this claim. Everything else is also unsupported and stands alone as premises. The second sentence most directly lends support to the conclusion. The third sentence mentions real life example of the situation described in the previous sentence, and provides a clear analogy to understand how the current healthcare system does not allow for actual cost reduction (push in on cost one place, it pops up somewhere else). The last sentence is just detailing an example of the reform.
Answer Choice (A) The tone of the passage certainly indicates that the author isn’t happy about shifting costs to the patient, but the argument also isn’t showing that shifting costs onto patients is contradictory; they are saying reducing costs under our system isn’t possible.
Answer Choice (B) The author doesn’t attribute fraudulent intent to anyone; he may not be happy about what the government and healthcare providers are doing, but he also is not saying they’re frauds or acting in fraudulent ways.
Correct Answer Choice (C) The argument is implying an analogy! The balloon air is an analogy for costs being passed off. This is how the argument tries to demonstrate the relationship of shifting, not reducing, costs between each party in healthcare.
Answer Choice (D) The author isn’t denying the possibility of a better system, he’s just saying that the one right now... ain’t it.
Answer Choice (E) There was no instance of cooperation in this stimulus.
We should recognize this is a strengthening question, since it asks: Which one of the following, if true, would most strengthen the argument?
Our stimulus begins by telling us that advertisers are often seen as unscrupulous (lacking restraint, essentially) in how they manipulate people’s desires. However, we’re then told that there is some evidence to the contrary; some advertisers are ethically motivated! That’s nice I guess, but I’m a bit skeptical; ‘ethical’ and ‘advertiser’ just don’t seem to go together often! Let’s see what evidence we have for this claim.
The argument cites a particular incidence where advertisers withdrew from a newspaper as it began to concentrate on sex and violence. Seems like the newspaper wants to profit from people’s desires! Our argument concludes the advertisers must have withdrew because they didn’t approve morally of the newspaper’s decision. This is the interpretation of the event which we want to support. What initially occurs to me is that it is entirely possible that the advertisers thought it would reflect badly on them if they were in the newspaper and could lose them money, and therefore their decision could have been entirely cynical. An answer choice which eliminates this alternate hypothesis that the advertisers were financially-motivated would be a good one. Let’s see what we get:
Answer Choice (A) Interesting, but this could be true if the advertisers withdrew for cynical reasons. This answer doesn’t give us enough information about the advertisers motives which is what we are interested in.
Answer Choice (B) We aren’t interested in those advertisers, we want to support a particular explanation of why some existing advertisers withdrew.
Correct Answer Choice (C) Bingo! If the advertisers knowingly took a haircut on their profits, that eliminates the alternate explanation that they did it all out of fear of losing money from not withdrawing.
Answer Choice (D) Ok? This, if anything, would lend support that the advertisers were motivated by financial rather than ethical considerations, and not strengthen our hypothesis.
Answer Choice (E) We don’t know whether this income group is poorer or richer and how that would affect the advertisers. Even if we did, how would it support our hypothesis that the advertisers withdrew because they were moral?
This is a strengthening question, as the stem asks us: Which one of the following, if true, supports the conclusion in the passage?
This is one of those questions where you can’t help but let out a sigh when you turn the page on your LSAT and it greets you. A long stimulus doesn’t necessarily mean a difficult question, but it usually means a time consuming one. We should always focus on sifting through the unnecessary details, and try to get a good grasp on the actual argument. This stimulus begins with a lot of context about computer software and how its standardization can make it vulnerable to viruses. Because computer software has become standardized, such that a business might have all its computers running on the same or similar software, a virus which infects one computer will have a high likelihood of being able to spread throughout the network to the other similar computers, giving a vandal the potential to destroy data on all the computers. Seems like a big problem! Luckily, our author has a solution; just make the software differ slightly between computers. By introducing minor variations that are compatible with business, the possibility of a virus destroying all data can be eliminate. This solution also wouldn’t entail any loss in computer compatibility. Our author concludes that this minor variation should be adopted. Our job is to strengthen this conclusion. Let’s see what the answer choices have in store for us:
Answer Choice (A) Interesting, but we’ve been explicitly told introducing minor variations won’t negatively impact this compatibility.
Correct Answer Choice (B) This strengthens our argument, since if true it would mean that whatever the costs associated with introducing minor variation are, they are worth avoiding the damage.
Answer Choice (C) Again, the author’s solution will not impact compatibility.
Answer Choice (D) Our conclusion is that a certain strategy will prevent a particular problem. The potential existence of other problems does nothing to strengthen the proposal.
Answer Choice (E) Interesting, but I don’t see how this would strengthen our proposed solution.